Difference between revisions of "1987 AJHSME Problems"

m (Problem 1)
(Problem 2)
Line 8: Line 8:
  
 
== Problem 2 ==
 
== Problem 2 ==
 +
 +
<math>\frac{2}{25}=</math>
 +
 +
<math>\text{(A)}\ .008 \qquad \text{(B)}\ .08 \qquad \text{(C)}\ .8 \qquad \text{(D)} 1.25 \qquad \text{(E)}\ 12.5</math>
  
 
[[1987 AJHSME Problems/Problem 2|Solution]]
 
[[1987 AJHSME Problems/Problem 2|Solution]]

Revision as of 15:01, 28 January 2009